Difference between revisions of "2021 AMC 10A Problems/Problem 3"

(Solution)
(Redirected page to 2021 AMC 12A Problems/Problem 3)
(Tag: New redirect)
 
(9 intermediate revisions by 3 users not shown)
Line 1: Line 1:
==Problem==
+
#redirect [[2021 AMC 12A Problems/Problem 3]]
These problems will not be released until the 2021 AMC 10A is released.
 
==Solution==
 
We don't know yet, but since 10+9=21, we know that the answer is G.
 
 
 
==Note==
 
This problem might also be on the AMC 12A. If so, please redirect it there.
 
==See also==
 
{{AMC10 box|year=2021|ab=A|num-b=2|num-a=4}}
 
{{MAA Notice}}
 

Latest revision as of 14:19, 11 February 2021